Difference between revisions of "2006 AMC 10A Problems/Problem 9"

 
(Redirect)
 
(4 intermediate revisions by 4 users not shown)
Line 1: Line 1:
== Problem ==
+
#redirect [[2006 AMC 12A Problems/Problem 8]]
How many sets of two or more consecutive positive integers have a sum of 15?
 
 
 
<math> \mathrm{(A) \ } 1\qquad \mathrm{(B) \ } 2\qquad \mathrm{(C) \ } 3\qquad \mathrm{(D) \ } 4\qquad \mathrm{(E) \ } 5 </math>
 
== Solution ==
 
== See Also ==
 
*[[2006 AMC 10A Problems]]
 

Latest revision as of 00:13, 28 April 2008